2016 AMC 10A Problems/Problem 14

Revision as of 18:56, 3 February 2016 by Warrenwangtennis (talk | contribs) (Created page with "==Solution== The amount of twos in our sum ranges from <math>0</math> to <math>1008</math>, with differences of <math>3</math> because <math>2 \cdot 3 = lcm(2, 3)</math>. Th...")
(diff) ← Older revision | Latest revision (diff) | Newer revision → (diff)

Solution

The amount of twos in our sum ranges from $0$ to $1008$, with differences of $3$ because $2 \cdot 3 = lcm(2, 3)$.

The possible amount of twos is $\frac{1008 - 0}{2} + 1 \rightarrow \boxed{\textbf{(B)} 337$ (Error compiling LaTeX. Unknown error_msg)